Exercise Sheet 7

61) Let \(P\) be the set of all divisors of \(12\). Determine the Möbius function of \((P,|)\) using the definition of the Möbius function and compare your result with the one from the last example in the lecture.

Hasse Hasse diagram for P={1,2,3,4,6,12} 12 12 4 4 12--4 6 6 12--6 2 2 4--2 6--2 3 3 6--3 1 1 2--1 3--1

The Möbius function is defined as follows:

\[\begin{gather*} \forall x,y \in P: \quad \sum_{z \in [x,y]} \varphi(z,y) = \begin{cases} &1 \textit{, if } x=y \\ &0 \textit{, if } x \neq y \end{cases} \end{gather*}\]

By definition \(\varphi(12,12) = 1\) and

\[\begin{gather*} \varphi(6,12) + \varphi(12,12) = 0 \end{gather*}\]

meaning \(\varphi(6,12) = -1\). Thus, by

\[\begin{gather*} \varphi(3,12) + \varphi(6,12) + \varphi(12,12) = 0 \end{gather*}\]

we derive \(\varphi(3,12) = 0\). And by

\[\begin{gather*} \varphi(4,12) + \varphi(12,12) = 0 \end{gather*}\]

we get that \(\varphi(4,12) = -1\). Now we have:

\[\begin{gather*} \varphi(2,12) + \varphi(4,12) + \varphi(6,12) + \varphi(12,12) ) = 0 \end{gather*}\]

from which we can derive \(\varphi(2,12) = 1\) Therefore, we know by:

\[\begin{gather*} \varphi(1,12) + \varphi(2,12) + \varphi(3,12) + \varphi(4,12) + \varphi(6,12) + \varphi(12,12) = 0 \end{gather*}\]

that \(\varphi(1,12) = 0\). The last example from the lecture states:

\[\begin{gather*} \varphi(1,n) = \varphi(n)= \varphi_{\geq}(0,e_1) \cdots \varphi_{geq}(0,e_r) \end{gather*}\]

where \(n = p_1^{e_1} \cdots p_r^{e_r}\) (prime factorisation of \(n\)).

\[\begin{gather*} \varphi(n)= \begin{cases} 1 &\text{, if } n=1\\ (-1)^r &\text{, if } n=p_1 \cdots p_r\\ 0 &\text{, if $n$ is not square free}. \end{cases} \end{gather*}\]

Since \(1 \cdot 2 \cdot 3 \cdot 4 \cdot 6 \cdot 12\) is not square free \(\varphi(12)=0\), which coincides with our findings.

62) Let \((P,|)\) be the poset defined by $P ={0,1,2,3,4} and \(0 \geq 1 \geq 4\), \(0 \geq 2 \geq 4\), \(0 \geq 3 \geq 4\). Compute all values \(\varphi(x,y)\) for \(x,y \in P\).

Hasse Hasse diagram for P={0,1,2,3,4} 0 0 4 4 2 2 4--2 3 3 4--3 1 1 4--1 2--0 3--0 1--0

We know:

\[\begin{gather*} \varphi(4,4) = \varphi(3,3) = \varphi(2,2) = \varphi(1,1) = \varphi(0,0) = 1 \end{gather*}\]

and

\[\begin{gather*} \varphi(0,1) = \varphi(0,3) = \varphi(0,2) = \varphi(1,4) = \varphi(2,4) = \varphi(3,4) = -1 \end{gather*}\]

and by the calculated values and

\[\begin{gather*} \varphi(0,4) + \varphi(1,4) + \varphi(2,4) + \varphi(3,4) + \varphi(4,4) = 0 \end{gather*}\]

we derive \(\varphi(0,4) = 2\).

63) Let \((P_1, \leq_1)\) and \((P_2, \leq_2)\) be two locally finite posets with \(0\)-element and \((P,\leq)\) be defined by \(P = P_1 \times P_2\) and for \((a,x),(b,y) \in P\):

\[\begin{gather*} (a,x) \leq (b,x) \Leftrightarrow a \leq_1 b \land x \leq_2 y. \end{gather*}\]

Show that \((P,\leq)\) is a locally finite poset with \(0\)-element.

A poset has to satisfy reflexivity, asymmetry and transitivity. We first show that \(P\) is a poset.

64) We use the notations from exercise 63. Let the Möbius functions of \(P,P_1,P_2\) be denoted by \(\varphi_P, \varphi_{P_1}, \varphi_{P_2}\), respectively. Prove that forall \((a,x) (b,y) \in P\) we have \(\varphi_P((a,x),(b,y)) = \varphi_{P_1} \cdot \varphi_{P_2}(x,y)\).

For \(\varphi_P((a,x),(b,y))\) is has to hold that:

\[\begin{gather*} \sum_{(c,z) \in [(a,x),(b,y)]} \varphi_{P((c,z),(b,y))} = \begin{cases} 1 &\text{ ,if $a=b$ and $x=y$}\\ 0 &\text{ ,if $a \neq b$ or $x \neq y$} \end{cases} \end{gather*}\]

The first condition holds, since:

\(\varphi_{P_1}(a,b) = 1\), when \(a=b\) and \(varphi_{P_2}(x,y) = 1\) when \(x = y\) thus \(\varphi_P((a,x),(b,y)) = 1 \cdot 1 = 1\), if \(a=b\) and \(x = y\).

We show, the second condition holds:

\[\begin{gather*} \sum_{(c,z) \in [(a,x),(b,y)]} \varphi_P((c,z),(b,y)) = \sum_{(c,z) \in [(a,x),(b,y)]} \varphi_{P_1}(c,b) \cdot \varphi_{P_2}(z,y) = \end{gather*}\]

we split the sum

\[\begin{gather*} \sum_{(c) \in [a,b]} \sum_{(z) \in [x,y]} \varphi_{P_1}(c,b) \cdot \varphi_{P_2}(z,y) \end{gather*}\]

we extract the constant factor \(\varphi_{P_1}(c,b)\)

\[\begin{gather*} \sum_{(c) \in [a,b]} \varphi_{P_1}(c,b) \cdot \sum_{(z) \in [x,y]} \varphi_{P_2}(z,y) \end{gather*}\]

Now either \(x \neq y\) or \(a \neq b\), w.l.o.g. we say \(x = y\)

\[\begin{gather*} \sum_{(c) \in [a,b]} \varphi_{P_1}(c,b) \cdot 0 = 0 \end{gather*}\]

65) Draw the Hasse diagram of \((2^{1,2,3}, \supseteq)\) and redo the proof of the principle of inclusion and exclusion for the special case of three sets \(A_1, A_2, A_3 \subseteq M\). Carry out every step in detail.

Hasse Hasse diagram for ($2^{1,2,3}$) 123 {1,2,3} 12 {1,2} 123--12 23 {2,3} 123--23 13 {1,3} 123--13 1 {1} 12--1 2 {2} 12--2 23--2 3 {3} 23--3 13--1 13--3 empty {} 1--empty 2--empty 3--empty

66) Let \(p,q,r\) be three distinct prime numbers and \(m = pqr\). How many of the numbers \(1,2, \dots, m\) are relatively prime to \(m\)? (Two numbers \(x\) and \(y\) are called relatively prime if their greatest common divisor is \(1\).)

\[\begin{gather*} m - ((\frac{m}{p} + \frac{m}{q} + \frac{m}{r}) - (\frac{m}{pq} + \frac{m}{pr} + \frac{m}{qr}) + \frac{m}{pqr}) = \end{gather*}\]

since, \(m = pqr\)

\[\begin{gather*} = m - (\frac{pqr}{p} + \frac{pqr}{q} + \frac{pqr}{r} - \frac{pqr}{pq} - \frac{pqr}{pr} - \frac{pqr}{qr} + \frac{pqr}{pqr}) = \end{gather*}\]

we cancel the respective factors in each term

\[\begin{gather*} = m - qr - pr - pq + r + q + p - 1 \end{gather*}\]

67) Prove: If \(gcd(a,b) = 1\) then \(gcd(a+b,a-b)\) is either \(1\) or \(2\).

Let \(d = gcd(a+b,a-b)\), then by \(\dagger\), \(d | (a+b)+(a-b)\), thus \(d|2a\), and \(d | (a+b)-(a-b)\), thus \(d | 2b\).

And since \(d|2a\) and \(d |2b\), \(d|gcd(2a,2b)\), by definition of the \(gcd\), and by \(gcd(a,b) = 1\), \(d\) can only either be \(1\) or \(2\).

Lemma \(\dagger\): Let \(z = gcd(x,y)\), then \(\exists m,n\), such that \(x=zm\) and \(y=zn\). Then \(x+y = zm + zn = z(m+n)\), thus \(z|(x+y)\) for arbitrary \(x,y\), where \(z = gcd(x,y)\) and \(x-y = zm - zn = z(m-n)\), thus \(z|(x-y)\) for arbitrary \(x,y\) such that \(z=gcd(x,y)\).

68) Let \(n = \prod_{p \in \mathbb{P}} p^{v_p (n)}\) be a positive integer such that for all \(p \in \mathbb{P}\) we have \(v_p(n) \leq 1\). Moreover, a prime \(p\) divides \(n\) if and only if \(p-1\) divides \(n\) too (\(\dagger\)). Compute \(n\).

We know that forall positive integers, \(1 | n\), thus by \(\dagger\) \(2 | n\), therefore also \(3|n\). Since \(2\) and \(3\) divide \(n\), \(6 | n\), hence by \(\dagger\), \(7 |n\). Then, \(n = 2 \cdot 3 \cdot 7 = 42\).

69) Use the Euclidian algorithm to find two integers such that:

\[\begin{gather*} 2863a + 1057b = 42 \end{gather*}\]

We first compute the division chain:

\[\begin{align} 2863 &= 1057 \cdot 2 + 749 \\ 1057 &= 749 \cdot 1 + 308 \\ 749 &=308 \cdot 2 + 133 \\ 308 &= 133 \cdot 2 + 42 \\ 133 &= 42 \cdot 3 + 7 \\ 42 &= 7 \cdot 6 + 0 \end{align}\]

We then find the linear combination for \(2863\) and \(1057\):

\[\begin{align} 42 &= 308 - (133 \cdot 2) \\ 42 &= 308 - ((749 - 308 \cdot 2) \cdot 2) = 308 \cdot 5 - 749 \cdot 2 \\ 42 &= (1057 - 749 \cdot 1) \cdot 5 - 749 \cdot 2 = 1057 \cdot 5 - 749 \cdot 7 \\ 42 &= 1057 \cdot 5 - (2863 - 1057 \cdot 2) \cdot 7 = 1057 \cdot 19 - 2863 \cdot 7 \end{align}\]

Thus we have found \(a = -7\) and \(b = 19\) which satisfy the equation.

70) Use the Euclidian algorithm to find all the greatest common divisors of \(x^3 + 5x^2 + 7x + 3\) and \(x^3 + x^2 - 5x +3\) in \(\mathbb{Q}[x]\).

\[\begin{align} x^3 + 5x^2 + 7x + 3 &= 1 \cdot (x^3 + x^2 - 5x + 3) + (4x^2 + 12x) \\ x^3 + x^2 - 5x + 3 &= (4x^2 + 12x) \cdot (1/4 x - 1/2) + (x+3) \\ 4x^2 + 12x &= (x+3) \cdot (4x) + 0 \end{align}\]

Thus, forall \(x \in \mathbb{Q}^x: a \cdot x + 3 \cdot a\) is a gcd of \(x^3 + 5x^2 + 7x + 3\) and \(x^3 + x^2 - 5x +3\).